Thread Rating:
  • 0 Vote(s) - 0 Average
  • 1
  • 2
  • 3
  • 4
  • 5
form1 q 26-35 - iqbalian
#1
26. A 27-year-old woman comes to the physician because of a 2-year history of intermittent diarrhea and severe cramping abdominal pain. The stools are watery, occasionally foul-smelling, and nonbloody. She is currently pain-free and has not had diarrhea for 2 days. She also has intermittent constipation. She has not had fever or weight loss. She returned from a trip to Mexico 3 months ago. She had an appendectomy at the age of 12 years and a cesarean delivery 4 years ago. Examination shows no abnormalities. Which of the following is the most likely diagnosis?

A) Bacterial gastroenteritis
B) Crohn's disease
C) Intermittent small-bowel obstruction
D) Irritable bowel syndrome
E) Laxative abuse

27. An asymptomatic 21-year-old woman is found to have an adnexal mass on pelvic examination. She uses oral contraceptives. A photograph of the mass is shown. Which of the following is the most likely diagnosis?

A) Benign cystic teratoma
B) Corpus luteum cyst
C) Dysgerminoma
D) Endometrioma
E) Mucinous cystoadenoma

28. A 16-year-old girl is brought to the physician because of episodes of palpitations over the past 6 months. The episodes occur when she runs or plays basketball. She is otherwise asymptomatic. Her blood pressure is 124/46 mm Hg, pulse is 78/min, and respirations are 18/min. She weighs 55 kg (121 lb) and is 180 cm (71 in) tall. Her arm span is 188 cm (74 in), and the upper segment to lower segment ratio is 0.85. Her fingers appear long and are hyperextensible. A grade 4/6, early diastolic murmur is heard along the upper and middle left sternal border with radiation to the apex. Peripheral pulses are bounding. Which of the following is the most likely cause of these findings?

a) Aortic incompetence
b) Aortic stenosis
C) Mitral incompetence
D) Mitral stenosis
E) Pulmonary incompetence
F) Pulmonary stenosis
G) Tricuspid incompetence
H) Tricuspid stenosis

29. A previously healthy 87-year-old woman comes to the physician because of a 4-month history of vulvar itching. Examination shows excoriated areas from scratching and a white, thin vulva. The labia minora are absent, and there are small fissures at the introitus. The remainder of the examination shows no abnormalities. Which of the following is the most likely diagnosis?

A) Escherichia coli infection
B) Lichen sclerosus
C) Squamous cell carcinoma
D) Trichomoniasis
E) Vulvar melanoma
F) Vulvar vestibulitis

30. One month after undergoing an uneventful renal transplant for chronic renal failure secondary to glomerulonephritis, a 38-year-old woman is hospitalized because of increased serum urea nitrogen (BUN) and creatinine levels. Prior to transplantation, she had been receiving hemodialysis for 3 years. Current medications include cyclosporine and prednisone. Examination shows no abnormalities. Over the past 48 hours, urine output has remained stable. Both renal biopsy and a radionuclide scan confirm the diagnosis of acute rejection. Which of the following is the most effective treatment?

A) Immediate discontinuation of cyclosporine
B) Increased dosage of corticosteroids
C) Diuresis and alkalinization of the urine
D) Renal dialysis for 1“2 weeks
E) Transplant nephrectomy

31. An 18-year-old man comes to the physician 1 week after he had a blood pressure of 140/110 mm Hg during a routine precollege examination. His temperature is 37.1 C (98.7 F), blood pressure is 140/100 mm Hg, pulse is 92/min, and respirations are 12/min. The upper extremities appear to be more muscular than the lower extremities. Radial pulses are normal; femoral, posterior tibial, and dorsalis pedis pulses are decreased. A grade 2/6 systolic murmur is heard over the precordium, anterior chest, and back. An ECG shows left ventricular hypertrophy. Which of the following is the most appropriate next step in management?
A ) Limiting physical activity
B ) Repeat blood pressure measurement in 1 month
C ) Initiate a low-sodium diet and exercise program
D ) Pharmacologic management
E ) Operative treatment

32. A 77-year-old woman comes to the physician because of a 2-day history of cramping abdominal pain and distention accompanied by nausea and vomiting. She is otherwise healthy and has no history of abdominal operations. Her temperature is 37.4 C (99.4 F), blood pressure is 110/86 mm Hg, pulse is 112/min, and respirations are 24/min. Cardiopulmonary examination shows no abnormalities. Examination of the abdomen shows distention and mild diffuse tenderness; bowel sounds are high-pitched. An x-ray film of the abdomen shows air-fluid levels throughout the small bowel and air in the liver; there is no gas in the colon or free air. Which of the following is the most likely diagnosis?

A) Adhesive small-bowel obstruction

B) Cecal cancer

C) Gallstone ileus

D) Intussusception

E) Mesenteric infarction

F) Ruptured appendicitis

G) Small bowel lymphoma

33. Four hours after undergoing a cesarean delivery at term followed by tubal ligation, a 37-year-old woman, gravida 2, para 2, has dizziness and confusion. The operation was uncomplicated, and blood loss is estimated to be 800 mL. Patient-controlled epidural analgesia has been moderately effective for pain. Her blood pressure now is 80/40 mm Hg, decreased from 120/72 mm Hg intraoperatively, and pulse is 152/min, increased from 96/min intraoperatively. Breath sounds are decreased bilaterally. No murmurs are heard. Abdominal examination shows distention and tenderness. Bowel sounds are absent. The incision is intact with no drainage. She is disoriented to person, place, and time. Her hematocrit is 23%; preoperative hematocrit was 35%. Which of the following is the most likely cause of the hemodynamic changes?

A) Epidural-related hypotension

B) Insufficient intraoperative fluid replacement

C) Postoperative intra-abdominal hemorrhage

D Supine hypotensive syndrome

E) Underestimated intraoperative blood loss

34. A 42-year-old woman comes to the physician for evaluation of persistently increased blood pressures. At her last two office visits during the past 3 months, her blood pressure has ranged between 150“170/105“115 mm Hg. During this period, she has had occasional headaches. In addition, she has had an increased urine output over the past 6 weeks that she attributes to a diet high in sodium. She is otherwise healthy and takes no medications. Her blood pressure today is 168/115 mm Hg, pulse is 68/min, and respirations are 14/min. Funduscopic examination shows mild arteriovenous nicking. The point of maximal impulse is not displaced. There is no edema, abdominal bruits, or masses. Serum studies show:

Na+
144 mEq/L
Cl“
90 mEq/L
K+
2.9 mEq/L
HCO3“
32 mEq/L
Urea nitrogen (BUN)
20 mg/dL
Creatinine
1.2 mg/dL

Which of the following is the most likely underlying cause of this patient's hypertension?
A) Autonomous production of aldosterone
B) Catecholamine-producing tumor
C) Decreased arterial distensibility caused by atherosclerosis
D) Excess production of atrial natriuretic peptide
E) Juxtaglomerular cell hypertrophy and sclerosis

35. A previously healthy 4-year-old girl is brought to the physician because of fever and refusal to walk for 1 day. She appears mildly ill. Her temperature is 38.6 C (101.5 F), pulse is 120/min, and respirations are 22/min. The right knee is erythematous and swollen. She holds her right knee in flexion and resists any attempted movement of her right leg. She cries when the right knee is moved. Which of the following is the most appropriate next step in management?

A) Acetaminophen with codeine therapy

B) Arthrocentesis

C) Bone marrow aspiration

D) Bone scan

E) Immobilization and traction

F) Lyme titer

G) MRI of the spine

H) Physical therapy

I) Reassurance

J) Serum rheumatoid factor assay

K) Systemic antibiotic therapy




plz comment on the answers

26)d
27)a
28)a
29b)
30)b
31)e???????
32)c
33)c
34a
35)b

Reply
#2
thank you everyone for not participating in discussing nbmes
Reply
#3
26-B
27-A
28-A
29-F
30-B
31-E
32-C
33-C
34-A
35-B
Reply
#4
HI qbalian
for Q26 should be Crohn's disease based on history of wt loss ,abdominal pain ,surgical history and chronic.
for Q 31 Diagnosis is coarction of aorta and treatment should be surgery
regarding 29 i am not sure
Reply
#5
iqbalian, thanks for all the questions, i just got from work and havent had time to answer but im sure everybody apreciates the questions and discussions. I agree with you on most of the answer
26. I believe is IBS, why Cronh's?
27. Dont have the picture.
28. Not good at murmur but A sounds pretty good to me, LOL
29.B
30.b
31.E
32.C
33.C
34. No idea, can anyone explain
35.B
Reply
#6
thanx for replying
26)can't be cronh's coz no h/o wt loss or bleeing in stools,fever
34)primary aldosteronism.no edem, polyuria and polydypsia,diatolic htn,hypokalemia
28 marfan synd,bounding pulses murmur radiating to axillla
Reply
#7
i am sorry for 26q answer is irrtable bowel syndrome
Reply
#8
hi iqbalian, thanks for the Qs, I agree with all your answers
Reply
#9
The murmur is not radiating to the axilla...
Reply
« Next Oldest | Next Newest »


Forum Jump: